HELP ME ASAP NO LINKS
Which student's expression results in a quotient of 12.5?
KWAN
VERA
16.25 : 1.3
- 11.25 = (-9)
Mark only one oval.
0000
a. Kwan's only
b. Vera's only
c. Both Kwan and Vera's
d. Neither Kwan nor Vera's

HELP ME ASAP NO LINKS Which Student's Expression Results In A Quotient Of 12.5?KWANVERA16.25 : 1.3- 11.25

Answers

Answer 1

Answer:

I wanna say its B. Vera's only

Answer 2
It should be B Vera’s only

Related Questions

QUICKLY!! We know that a triangle with side lengths x^2-1,2x and x^2+1 is a right triangle. Using those side lengths, find the missing triples and x-values.


Write the triples in parentheses, without spaces between the numbers, and with a comma between numbers. Write the triples in order from least to greatest.


Type the correct answer in each box.



x value--------------------pythagorean triple

3 ____________



________ (8,15,17)


5 ______________



__________ (12,35,37)

Answers

Answer:

[tex]\begin{array}{ccl}x \ value&&Pythagorean \ triple\\3&&(6, 8, 10)\\4&&(8, 15, 17)\\5&&(10, 24, 26)\\6&&(12, 35, 37)\end{array}[/tex]

Step-by-step explanation:

The given side lengths of the right triangle are;

x² - 1, 2·x and x² + 1

A Pythagorean triple are three numbers, a, b, and c, such that, we have;

a² + b² = c²

From the given side lengths, we have;

We note that (x² + 1) > (x² - 1)

(x² + 1) > 2·x for x > 1

Therefore, with (x² + 1) as the hypotenuse side, we have;

(x² - 1)² + (2·x)² = (x² + 1)²

Therefore, when the x-value is 3, we have;

(3² - 1)² + (2 × 3)² = (3² + 1)²

8² + 6² = 10²

The least is 6² = (2 × 3)², from (2·x)²

Therefore;

The Pythagorean triple is 6, 8, 10

The order of the triple is (2·x), (x² - 1), (x² + 1)

2) The x-value for the triple, (8, 15, 17), is obtained as follows;

The least, 8 = 2·x

∴ x = 8/2 = 4

The x-value = 4

3) The Pythagorean triple where the x-value = 5 is therefore;

(2·x), (x² - 1), (x² + 1), where x = 5 gives;  (2×5 = 10), (5² - 1 = 24), (5² + 1 = 26)

Therefore, the Pythagorean triple where x = 5 is 10, 24, and 26

4) The x-value for the Pythagorean triple (12, 35, 37) is given by 12 = 2·x

Therefore, x = 12/2 = 6

Therefore, we get;

[tex]\begin{array}{ccl}x \ value&&Pythagorean \ triple\\3&&(6, 8, 10)\\4&&(8, 15, 17)\\5&&(10, 24, 26)\\6&&(12, 35, 37)\end{array}[/tex]

An online store increased the price of a shirt by 17% and charged $3 to ship the shirt to a customer. The customer paid $43 for the shirt. What was the original price of the shirt?


Please explain thoroughly how to get the answer. 15 points

Answers

Answer:

Check explanation

Step-by-step explanation:

Let

x = original price

Percentage increase in price = 17%

New price = original price + Percentage increase in price

= x + 17% of x

= x + 0.17 * x

= x + 0.17x

= 1.17x

Shipping cost = $3

Total cost = $43

Total cost = New price + Shipping cost

43 = 1.17x + 3

43 - 3 = 1.17x

40 = 1.17x

x = 40 / 1.17

x = $34.188034188034

Approximately,

x = $34.2

2+3=95
4+5=259
6+7=4913
8+9=?​

Answers

Answer:

8117

Step-by-step explanation:

by considering  2 + 3 = 95

3² = 9

2 + 3 = 5      

by considering  4 + 5 = 259

5² = 25

4 + 5 = 9  

by considering  6 + 7 = 4913

7² = 49

6 + 7 = 13  

by considering  8 + 9 = ????

9² = 81

8 + 9 = 17  

8 + 9 = 8117

2m^2-5m-3=0 by factorization​

Answers

Step-by-step explanation:

It is so simple Hope u understand

Answer:

Step-by-step explanation:

Sum = -5

Product = 2*(-3) = -6

Factors = -6 , 1   {-6 + 1 = -5  & -6 *1 = -6}

2m² - 5m -3 = 0

2m² - 6m + m -3 = 0

2m(m - 3) + (m -3) = 0

(m -3)(2m + 1) = 0

m - 3 =  0    or 2m + 1 = 0

    m = 3      or 2m = -1

                          m = -1/2

Ans: m = 3 , (-1/2)

Help plsssssss ,it would mean a lot thankyou

Answers

Answer:

Part 1;

(0, 0)

Part 2;

(0, 2.5)

Step-by-step explanation:

Part 1

The given system of inequalities is presented as follows;

f(x) < x + 4; f(x) > -x - 3; and f(x) < 5

We check each of the points as follows;

The point (0, 0) in the inequality f(x) < x + 4, gives;

f(0) < 0 + 4

f(0) = 0 < (is less than) 0 + 4 = 4

Therefore, (0, 0) is a solution of the inequality f(x) < x + 4

The point (0, 0) in the inequality f(x) > -x - 3, gives;

f(0) < -0 - 3

f(0) = 0 > (is larger than) -0 - 3 = -3

Therefore, (0, 0) is a solution of the inequality f(x) > -x - 3

The point (0, 0) in the inequality f(x) < 5, gives;

f(0) < 5

f(0) = 0 < (is less than) 5

Therefore, (0, 0) is a solution of the inequality f(x) < 5

The point (-6, 0) in the inequality f(x) > -x - 3, gives;

f(-6) = -6 - 3 = 3

The point (-6, 0) with y = 0 < (is less than) f(-6) = 3, therefore (-6, 0) is not a solution to (not included in the graph of) the inequality f(x) > -x - 3 and therefore to the system of inequalities because at x = -6, the values of f(x) > -x - 3 are larger than 3

The point (-3, 4) in the inequality f(x) < x + 4, gives;

f(-3) = -3 + 4 = 1

The point (-3, 4) with y = 4 < (is larger than) f(-2) = 1, therefore (-3, 4) is not a solution to (not included in the graph of) the inequality f(x) < x + 4  and therefore to the system of inequalities because at x = -3, the values of f(x) < x + 4 are less than 1

The point (4, 6) is not a solution to (not included in the graph of) the inequality f(x) < 5  and therefore to the system of inequalities because f(x) is larger than 5 for all x

Therefore, the point which is part of the solution set of the system of inequalities is (0, 0)

Part 2

The given system of inequalities are, f(x) ≥ 2·x + 2; f(x) ≤ -4·x + 3; and f(x) ≤ 6·x + 5

Plotting the given system of inequalities on MS Excel the point which is part of the solution is given by points which are within the triangular intersection area of the three inequalities, with coordinates, (1/6, 7/3), (-3/4, 1/2), and (-1/5, 19/5)

Therefore, the points, (0, 0), (-2.5, 0) are not solutions because, the y-value of the solution area are all higher than the line y = 0

The point (0. 6.5) is not a solution because the points in the triangular solution area all have x-values lesser than x = 6.5

The point which is part of the solution by examination is the point (0, 2.5) which is a point between the lines y = 19/5 = 3.8, y = 1/2, x = -3/4, and x = 1/6.

How do I answer number 1

Answers

Answer:

#1 Haley is correct and Lacey is incorrect

#2 Kenji is incorrect.

Step-by-step explanation:

#1. x^3 (x^2) = x^5 but this same law doesn't apply to addition of numbers with exponents.

#2 The law of exponents doesn't apply to numbers with different bases that are not multiples of each other such as 3 and 4, so Kenji's simplification is not correct.

Type the correct answer in each box.
Bridget went fishing with her dad. Bridget caught the first fish of the day, and it weighed f ounces. That day, she caught four more fish. One was
2
times the weight of the first fish, another was
2
more than
3
times the weight of the first fish, the next was
1
2
the weight of the first fish, and the last was
3
5
the weight of the first fish.

Bridget’s dad caught four fish. The first fish he caught weighed
2
more than
3
times the weight of the first fish caught that day.

One fish weighed
4
5
the weight of the first fish caught that day, one weighed
4
more than
2
times the weight of the first fish caught that day, and the last weighed
1
2
the weight of the first fish caught that day.

Answers

Answer:

PLZZ MARK ME BRAINLIEST..!

Step-by-step explanation:

Bridgets fish: f , 2f, 3f+2 , 1/2f, 3/5f

Add for total weight: 7 1/10 f +2

Dads fish: 3f+2, 4/5f, 2f+4, 1/2f

Add for total weight:  6 3/10f +6

set the 2 total weights equal:

6 3/10f +6 = 7 1/10f +2

Subtract 6 3/10f from each side:

6 = 8/10f  + 2

Subtract 2 from each side:

4 = 8/10f

Divide both sides by 8/10:

f =  5

Bridget's first fish weighed 5 ounces.

Dads first fish weighed:  2 more than 3 times :3(5) + 2 = 15 +2 = 17 ounces.

[tex]\text{Solve for x.}\\\\5x + 10 = 35[/tex]

Answers

Answer:

x = 5

Step-by-step explanation:

5x + 10 = 35

Subtract 10 from both sides

5x + (10-10) = 35 - 10

Simplify

5x = 25

Divide both sides by 5

5x/5 = x

25/5 = 5

We're left with x = 5

Hi there!  

»»————- ★ ————-««

I believe your answer is:  

[tex]\boxed{x = 5}[/tex]

»»————- ★ ————-««  

Here’s why:  

⸻⸻⸻⸻

[tex]\boxed{\text{Solving for 'x'....}}\\\\5x + 10 = 35\\-------------\\\rightarrow 5x + 10 - 10 = 35 - 10\\\\\rightarrow 5x = 25\\\\\rightarrow \frac{5x=25}{5}\\\\\rightarrow \boxed{x = 5}[/tex]

⸻⸻⸻⸻

»»————- ★ ————-««  

Hope this helps you. I apologize if it’s incorrect.  

Solve for x. Round to the nearest tenth, if necessary.

Answers

I think x= 0.6 because

Sin60/1 = sin30/x
Sin30 x 1/sin60
X= 0.6

find the 9th and 15th terms of the following geometric sequence 2, -4, 8, -16​

Answers

Step-by-step explanation:

given the geometric sequence 2, -4, 8, -16, ...

a1 = 2

r = -4/2 = -2

find : a9 and a15

solutions:

an = a1. r^(n-1)

=> a9 = 2. (-2)^(9-1)

= 2. (-2)^8

= 2. 2^8

= 2^9

= 512.

=> a15 = 2. (-2)^(15-1)

= 2. (-2)^14

= 2. 2^14

= 2^15

= 32,768

Step-by-step explanation:

Hey there!

The given geometric sequence is: 2, -4, 8, -16.

The;

a1 = 2

Common ratio (r) = T2/T1

= -4/2

= -2

Now;

Use general formula of geometric sequence;

[tex]tn = {a1.r}^{n - 1} [/tex]

Where, "a1" is first term, "n" is no.of terms and "r" is common ratio.

Then;

[tex]t9 = 2 \times { (- 2)}^{9 - 1} [/tex]

or, t9 = 2*256

Therefore, t9 = 512.

Again;

[tex]t15 = 2. { (- 2)}^{15 - 1} [/tex]

or, t15= 2*16384

Therefore, t15 = 32768.

Hope it helps!

Variables, in statistics, refer to:
A) characteristics of experimental units
B) data that has been collected
C) unknown quantaties

Answers

the answer to this question is a

There are no results for an 8 sideded fair die with faces labelled 2,3,3,4,7, 7,7,7 and 9 is rolled finmd the probibilty sou;ltio

Answers

Answer:

Probability of getting a number which is not 2 = 7/8

Step-by-step explanation:

Given;

Total number of faces = 8

Number of face with 2 = 1

Number of face with 3 = 2

Number of face with 4 = 1

Number of face with 7 = 3

Number of face with 9 = 1

Find:

Probability of getting a number which is not 2

Computation:

Probability of not an event = 1 - [Number of favourable outcomes / Total number of outcomes]

Probability of getting a number which is not 2 = 1 - [1/8]

Probability of getting a number which is not 2 = [8-1] / 8

Probability of getting a number which is not 2 = 7/8

The graph of a line goes down and to the right when:
A. there is no coefficient of x.
B. the coefficient of x is 0.
c. the coefficient of xis positive.
D. the coefficient of x is negative.

Answers

Answer:

The answer is D.

Step-by-step explanation:

When a graph of a line goes down and to the right, it shows that as the value of x increases, the value of y decreases. This represents a negative coefficient for x.

The graph is a drawn out version of the question.

Te graph of a line goes down and to the right when the coefficient of x is negative, option (D) is correct.

What is a straight line?

A straight line is a combination of endless points joined on both sides of the point.

The slope 'm' of any straight line is given by:

[tex]\rm m =\dfrac{y_2-y_1}{x_2-x_1}[/tex]

y = mx + c

We have:

The graph of a line goes down and to the right when.

The orientation of the line depends on the slope.

If the slope of the line is positive, the line will go up.

If the slop of the line is negative, the line will go down.

Thus, the graph of a line goes down and to the right when the coefficient of x is negative, option (D) is correct.

Learn more about the straight line here:

brainly.com/question/3493733

#SPJ2

What is the mean change in the forecasted low temperatures over the next 7 days? Remember, this can be found by averaging the values in the Difference column for the low temperatures. If your answer is not an integer, explain what two integers your answer is between.
LOW TEMPTURES:
WEDNESDAY-79
THURSDAY- 79
FRIDAY- 73
SATURDAY- 73
SUNDAY - 73
MONDAY- 73
TUESDAY- 75

Answers

Answer:

75

Step-by-step explanation:

Which of the four graphs has the greatest standard deviation? please help me ​

Answers

Answer:

Standard deviation is how far away the values are from the mean.All of your graphs have normal distribution, meaning the mean is in the center.

The more spread out your graph is, the greater the standard deviation.

So the option with the most spread out graph, which I think is A, can't see very clear.

Which classification best represents a triangle with side lengths 6 cm, 10 cm, and 12 cm?

acute, because 62 + 102 < 122
acute, because 6 + 10 > 12
obtuse, because 62 + 102 < 122
obtuse, because 6 + 10 > 12

Answers

Answer:

C

Step-by-step explanation:

use Pythagorean theorem

[tex]a^{2}[/tex] + [tex]b^{2}[/tex] = [tex]c^{2}[/tex]

c is the longest side

if [tex]a^{2}[/tex] + [tex]b^{2}[/tex] > [tex]c^{2}[/tex] then it's acute (greater than)

if [tex]a^{2}[/tex] + [tex]b^{2}[/tex] < [tex]c^{2}[/tex] then it's obtuse   (less than)

if they are equal, then its a right triangle

[tex]6^{2}[/tex] + [tex]10^{2}[/tex] = [tex]12^{2}[/tex]

36 + 100 = 144

136 = 144

136 < 144   obtuse

The correct classification for this triangle is:

obtuse, because 6² + 10² < 12²

Option C is the correct answer.

What is a triangle?

A triangle is a 2-D figure with three sides and three angles.

The sum of the angles is 180 degrees.

We can have an obtuse triangle, an acute triangle, or a right triangle.

We have,

To determine the classification of a triangle based on its side lengths, we can use the triangle inequality theorem, which states that the sum of the lengths of any two sides of a triangle must be greater than the length of the third side.

In this case, we have a triangle with side lengths of 6 cm, 10 cm, and 12 cm. Checking the sum of the lengths of each pair of sides, we have:

6 + 10 = 16 > 12

6 + 12 = 18 > 10

10 + 12 = 22 > 6

Since all three pairs satisfy the triangle inequality theorem, the given side lengths do form a valid triangle.

Next, we can use the law of cosines to determine the measure of the largest angle in the triangle, which will allow us to classify it.

The law of cosines states that, for a triangle with side lengths a, b, and c, and the angle opposite c denoted as C, we have:

[tex]c^2 = a^2 + b^2 - 2ab cos(C)[/tex]

In this case, the side lengths are a = 6 cm, b = 10 cm, and c = 12 cm. Substituting these values into the formula and solving for cos(C), we get:

cos(C) = (6² + 10² - 12²) / (2 x 6 x 10)

cos(C) = -1/5

Since the cosine function is negative for angles between 90 and 180 degrees, we know that angle C is obtuse.

Therefore,

The correct classification for this triangle is:

obtuse, because 6² + 10² < 12²

Learn more about triangles here:

https://brainly.com/question/25950519

#SPJ7

Find the value of x.
A. About 57.6
B. About 42.6
C. About 12.6
D. About 27.6

Answers

Answer:

about 27.6

Step-by-step explanation:

The sum of interior angles for this rectangle is 1080

119+140+124+6x+132+132+102 = 1080 add like terms

749 + 12x = 1080 subtract 749 from both sides

12x = 331 divide both sides by 12

x = 27.6 approximately

A(3,4) and B(-3,2) are pointd on a coordinate plane. find the coordinate of a points C on the x axis such that AC=BC​

Answers

Answer:

Step-by-step explanation:

Here's the game plan. In order to find a point on the x-axis that makes AC = BC, we need to find the midpoint of AB and the slope of AB. From there, we can find the equation of the line that is perpendicular to AB so we can then fit a 0 in for y and solve for x. This final coordinate will be the answer you're looking for. First and foremost, the midpoint of AB:

and

Now for the slope of AB:

and

 So if the slope of AB is 1/3, then the slope of a line perpendicular to that line is -3. What we are finding now is the equation of the line perpendicular to AB and going through (0, 3):

and filling in:

y - 3 = -3(x - 0) and

y - 3 = -3x + 0 and

y - 3 = -3x so

y = -3x + 3. Filling in a 0 for y will give us the coordinate we want for the x-intercept (the point where this line goes through the x-axis):

0 = -3x + 3 and

-3 = -3x so

x = 1

The coordinate on the x-axis such that AC = BC is (1, 0)

Let a and b be the solutions to x^2 + x − 3 = 0. Find the value of a^3 − 4b^2 + 19.
If you can't solve it don't answer.
This is a challenge.
Good luck!

Answers

Answer:

0.037

Step-by-step explanation:

Given that,

Let a and b be the solutions to [tex]x^2 + x -3 = 0[/tex]

It can be solved using quadratic formula where a = 1, b = 1 and c = -3

So,

[tex]x=\dfrac{-1+\sqrt{1^2-4\times 1\times (-3)}}{2(1)},\dfrac{-1-\sqrt{1^2-4\times 1\times (-3)}}{2(1)}\\\\x=1.30,-2.3[/tex]

Let a = 1.3, b = -2.3

The value of [tex]a^3 -4b^2 + 19[/tex] can be given by :

[tex]a^3 -4b^2 + 19=(1.3)^3-4\times (-2.3)^2+19\\\\=0.037[/tex]

So, the value of the given expression is 0.037.

question 2 help pls

Answers

x=-2
5-4=1
so whatever is square rooted must be equal to 1
1 squared is 1
-2+3 is one

Answer:

[tex](\sqrt{x+3} )^{2} + 4^{2} = 5^{2}[/tex]

x + 3 + 16 = 25

x + 19 = 25

x = 6

hope it helps.

Also, I think that Brainly is an awesome app, but there's an app which is doing great work for me in maths, named Gauthmath. I will suggest it. Video concepts and answers from real tutors.

I need help with my math!!!

Answers

Answer:

The correct answer is y = | x + 6 |

65. A city has a population of 25,000. The population is expected to increase by 5.5% annually for the
next decade. (See Example 5)
a. Write a function that represents the
City Population
population y after t years.
УГ
40,000
35,000
30,000
25,000
b. Graph the function from part (a). Use the 20,000
graph to estimate the population after 4
15,000
years.
10,000
5000
0
0 1 2 3 4 5 6 7 8 t
Year
Population

Answers

Answer:

The answer will be 0.45%

Step-by-step explanation:

im right

Amy,bob and hadi saved $1012 altogether. Amy saved $85. Hadi saved 3 times as much as the total amount of Amy's and Bob's savings. How much did Bob save?

Answers

Answer:

Bob Saves 168$.  

Step-by-step explanation:

According to the Question,

Given That,   Amy , bob and hadi saved $1012 altogether.

Thus, A + B + H = 1012 ⇒ Amy saved $85.

So, B + H = 927 ----- Equation 1

And Hadi saved 3 times as much as the total amount of Amy's and Bob's savings So, H = 3(A + B) ⇒ We Know Amy saved $85.

So, H - 3B = 255  ----- Equation 2

Now, Subtract Equation 2 from Equation 1, We get

4B = 672 ⇒ B = 168

Bob Saves 168$.

You are a salaried employee paid semi-monthly. If you make 548,750 annually, what is
your estimated take home per pay?

Answers

If semi-monthly means every half a month, then my take home pay is 22864.58333

Answer:

548750/12

approximately 45729

Step 1: –10 + 8x < 6x – 4
Step 2: –10 < –2x – 4
Step 3: –6 < –2x
Step 4: ________

What is the final step in solving the inequality –2(5 – 4x) < 6x – 4?

x < –3
x > –3
x < 3
x > 3Step 1: –10 + 8x < 6x – 4
Step 2: –10 < –2x – 4
Step 3: –6 < –2x
Step 4: ________

What is the final step in solving the inequality –2(5 – 4x) < 6x – 4?

x < –3
x > –3
x < 3
x > 3

Answers

x< 3 because whenever you multiply or divide by a negative number, the symbol switches.

You offer to sell a used car for $1,895. Yesterday you purchased the car for $1,755. What percentage markup on cost are you charging (to the nearest tenth)?

Answers

Answer:

8.0%

Step-by-step explanation:

1. [tex]1895-1755=140[/tex]

2.[tex]\frac{140}{1755} =0.07977208[/tex]

3. 8.0%

❊ Simplify :
[tex] \large{ \bf{ \frac{x - 1}{ {x}^{2} - 3x + 2} + \frac{x - 2}{ {x}^{2} - 5x + 6 } + \frac{x - 5}{ {x}^{2} - 8x + 15 } }}[/tex]
[tex] \large{ \tt{ans : \bf{ \frac{3x - 7}{(x - 2)(x - 3)} }}}[/tex]
- Show your workings *
- Irrelevant / Random answers will be reported!​

Answers

Need to Do :- To simplify the given expression .

[tex]\red{\frak{Given}}\Bigg\{ \sf \dfrac{x - 1}{ {x}^{2} - 3x + 2} + \dfrac{x - 2}{ {x}^{2} - 5x + 6 } + \dfrac{x - 5}{ {x}^{2} - 8x + 15 } [/tex]

[tex]\rule{200}4[/tex]

[tex]\sf\longrightarrow \small \dfrac{x - 1}{ {x}^{2} - 3x + 2} + \dfrac{x - 2}{ {x}^{2} - 5x + 6 } + \dfrac{x - 5}{ {x}^{2} - 8x + 15 } \\\\\\\sf\longrightarrow \small \dfrac{ x-1}{x^2-x -2x +2} +\dfrac{ x-2}{x^2-3x-2x+6} +\dfrac{ x -5}{x^2-5x -3x + 15 } \\\\\\\sf\longrightarrow\small \dfrac{ x -1}{ x ( x - 1) -2(x-1) } +\dfrac{ x-2}{x ( x -3) -2( x -3)} +\dfrac{ x -5}{ x(x-5) -3( x -5) } \\\\\\\sf\longrightarrow \small \dfrac{ x -1}{ ( x-2) (x-1) } +\dfrac{ x-2}{( x -2)(x-3) } +\dfrac{ x -5}{ (x-3)(x-5) } \\\\\\\sf\longrightarrow\small \dfrac{ 1}{ x-2} +\dfrac{ 1}{ x -3} +\dfrac{1}{ x -3} \\\\\\\sf\longrightarrow \small \dfrac{1}{x-2} +\dfrac{2}{x-3} \\\\\\\sf\longrightarrow \small \dfrac{ x-3 +2(x-2)}{ ( x -3)(x-2) } \\\\\\\sf\longrightarrow \small \dfrac{ x - 3 +2x -4 }{ (x-3)(x-2) } \\\\\\\sf\longrightarrow \underset{\blue{\sf Required \ Answer }}{\underbrace{\boxed{\pink{\frak{ \dfrac{ 3x -7}{ ( x -2)(x-3) } }}}}}[/tex]

[tex]\rule{200}4[/tex]

Answer:

Your solution ..................

how do you get sin theta​

Answers

Sin=opp/hyp
And 1/csc
And root 1-cos^2


The pie chart shows how 36 pupils travel to school.
Use the pie chart to complete the table.
Bike
Walk
Travel to
school
Number
of pupils
Walk
9
120°
Car
Bus
Bus
Car
Bike
Total
36

Answers

Answer:

[tex]Bike = 7[/tex]

[tex]Car = 8[/tex]

[tex]Walk = 9[/tex]

[tex]Bus = 12[/tex]

Step-by-step explanation:

Given

[tex]Bus = 120^o[/tex]

[tex]Walk = 90^o[/tex]

[tex]Car = 80^o[/tex]

[tex]n = 36[/tex] --- pupils

Required

Determine the number of students in each category

This is calculated by dividing the measure of each category by 360; then multiply the result by the number of pupils:

So, we have:

[tex]Bus = \frac{120}{360} * 36 = \frac{1}{3} * 36 = 12[/tex]

[tex]Walk = \frac{90}{360} * 36= \frac{1}{4} * 36 = 9[/tex]

[tex]Car = \frac{80}{360} * 36 = \frac{80}{10} = 8[/tex]

To calculate the number of students that travel by bike, we have:

[tex]Car + Bike + Walk + Bus= n[/tex]

Substitute values

[tex]8 + Bike + 9+ 12= 36[/tex]

Collect like terms

[tex]Bike = 36 - 8 - 9 - 12[/tex]

[tex]Bike = 7[/tex]

Answer:

Look at picture

Step-by-step explanation:

f equals to 2 f - 20​

Answers

Answer:

20

Step-by-step explanation:

f = 2f - 20

f - 2f = - 20

- f = - 20

f = 20

Other Questions
Because of vast differences between different regions, multinational corporations often use ______ departmentalization. how to Write a simple Java socket programming that a client sends a text and server receives and print. Skill-Related Element Activity ExplanationAgility Balance Coordination Power Reaction Time Speed help me with 2 excersise , thanks a lot help please what is the median 8. When someone buys a ticket for an airline flight, there is a 0.0995 probability that the person will not show up for the flight. An agent for Air America wants to book 24 people on an airplane that can seat only 22. If 24 people are booked, find the probability that not enough seats will be available. You must show your work using a calculator function to receive credit. Which is a source of adult stem cells?bone marrowamniotic fluidembryosgametes (a) Find the average rate of change of the area of a circle with respect to its radius r as r changes from 2 to each of the following.(i) 2 to 3 (ii) 2 to 2.5 (iii) 2 to 2.1(b) Find the instantaneous rate of change when r =2. Type the correct answer in each box. Use numerals instead of words.Consider this expression.- 4x2 + 2x - 5(1 + x)What expression is equivalent to the given expression? Which sentence states implicit information you can learn from the passage? what causes pressure inside a bicycle tire Please help me solve this problem Im really struggling Which 2 resonance forms destablize the carbocation intermediate if bezonitrile undergoes chlronation at the ortho or para positions 20. (06.07 )The graph below plots the values of y for different values of20151055Which correlation coefficient best matches the data plotted on the graph? (1 point)-0.500.250.90 Present perfect tense for Ryan has six cats and four dogs Write a letter to a friend describing a difficult situation you are going through at the moment. A bullet fired vertically at a velocity of 36m/s .after 45 the bullet hit the top of a bulid how height is a bulid? In a 45-45-90 right triangle, what is the ratio of the length of one leg to the length of the other leg? 17. Which of the following is correct? a. b. He have a lot of money. You has a lot of money. I have a lot of money. all of the above C. d. According to the prompt, how will you support your analysis?